9310855720

Enroll Now
Student Login
Enroll Now
Student Login

9310855720

UPSC  GS Prelims Solved Paper 2024 : Set C
Home >   Prelims

2024: Set C

Solved Questions with Answer

1. Consider the following statements:

Statement-I:

There is instability and worsening security situation in the Sahel region.

Statement-II:

There have been military takeovers/coups d'état in several countries of the Sahel region in the recent past.

Which one of the following is correct in respect of the above statements?

(a) Both Statement-I and Statement-II are correct and Statement-II explains Statement-I

(b) Both Statement-I and Statement-II are correct, but Statement-II does not explain Statement-I

(c) Statement-I is correct, but Statement-II is incorrect

(d) Statement-I is incorrect, but Statement-II is correct

Show Answer

Ans: (a)

Explanation

Statement I is correct: The Sahel region, which includes countries like Mali, Burkina Faso, Niger, and Chad, has been facing a worsening security situation in recent years. This instability is driven by the rise of violent extremist groups

Statement II is correct: The instability in the Sahel has also led to a series of military coups in several countries in the region, for example, there have been military takeovers in Mali, Chad, Guinea, and Burkina Faso in recent years. These coups are a direct consequence of the worsening security situation, as the military has often stepped in to take control in response to the inability of civilian governments to address the security challenges

2. Consider the following statements :

Statement-I:

India does not import apples from the United States of America.

Statement-II:

In India, the law prohibits the import of Genetically Modified food without the approval of the competent authority.

Which one of the following is correct in respect of the above statements?

(a) Both Statement-I and Statement-II are correct and Statement-II explains Statement-I

(b) Both Statement-I and Statement-II are correct, but Statement-II does not explain Statement-I

(c) Statement-I is correct, but Statement-II is incorrect

(d) Statement-I is incorrect, but Statement-II is correct

Show Answer

Ans: (d)

Explanation

Statement I is incorrect: India imports apples from the United States. The import of apples from the US has been significant in the past, with the country being one of the major apple exporters to India. However, the import volume had decreased significantly due to the imposition of retaliatory tariffs by India in 2019.

Statement II is correct: India has regulations in place regarding the import of Genetically Modified Organisms (GMOs) including food and agricultural products. The import of GMOs requires approval from the Genetic Engineering Appraisal Committee (GEAC) under the Ministry of Environment, Forest and Climate Change.

3. With reference to the Speaker of the Lok Sabha, consider the following statements:

While any resolution for the removal of the  Speaker of the Lok Sabha is under consideration

1. He/She shall not preside.

2. He/She shall not have the right to speak.

3. He/She shall not be entitled to vote on the resolution in the first instance.

Which of the statements given above is/are correct?

(a) 1 only

(b) 1 and 2 only

(c) 2 and 3 only

(d) 1, 2 and 3

Show Answer

Ans: (a)

Explanation

Statement 1 is correct: According to Article 94(c) of the Indian Constitution and Rule 179 of the Rules of Procedure and Conduct of Business in Lok Sabha, when a resolution for the removal of the Speaker is under consideration, the Speaker shall not preside over the House.

Statement 2 is incorrect: He/She shall not have the right to speak: The Speaker retains the right to speak and defend themselves against the charges.

Statement 3 is incorrect: He/She shall not be entitled to vote on the resolution in the first instance: The Speaker is entitled to vote on the resolution but only in the case of a tie, as per the general practice of voting rights of the Speaker in the Lok Sabha.

4. With reference to the Indian Parliament, consider the following statements:

1. A bill pending in the Lok Sabha lapses on its dissolution.

2. A bill passed by the Lok Sabha and pending in the Rajya Sabha lapses on the dissolution of the Lok Sabha.

3. A bill in regard to which the President of India notified his/her intention to summon the Houses to a joint sitting lapses on the dissolution of the Lok Sabha.

Which of the statements given above is/are correct?

(a) 1 only

(b) 1 and 2

(c) 2 and 3

(d) 3 only

Show Answer

Answer: ( b)


5. With reference to the Parliament of India, consider the following statements:

1. Prorogation of a House by the President of India does not require the advice of the Council of Ministers.

2. Prorogation of a House is generally done after the House is adjourned sine die but there is no bar to the President of India prorogating the House which is in session.

3. Dissolution of the Lok Sabha is done by the President of India who, save in exceptional circumstances, does so on the advice of the Council of Ministers.

Which of the statements given above is/are correct?

(a) 1 only

(b) 1 and 2

(c) 2 and 3

(d) 3 only

Show Answer

Ans: (c)

Explanation

Statement 1 is incorrect: The prorogation of a House by the President of India requires the advice of the Council of Ministers. According to Article 74 of the Indian Constitution, the President shall act in accordance with the advice of the Council of Ministers headed by the Prime Minister. Therefore, the President cannot unilaterally prorogue a House without this advice.

Statement 2 is correct: Prorogation is typically done after the House is adjourned sine die, but the President can prorogue the House while it is in session. Article 85(2)(a) of the Constitution allows the President to prorogue either House of Parliament at any time, irrespective of whether the House is adjourned sine die or still in session.

Statement 3 is correct: The dissolution of the Lok Sabha is usually done by the President on the advice of the Council of Ministers. Article 85(2)(b) of the Constitution states that the Lok Sabha shall be dissolved by the President. This action is generally taken on the advice of the Council of Ministers, reflecting the practice in a parliamentary democracy. In exceptional circumstances, such as a political crisis or no-confidence motion, the President may act without such advice, exercising discretion to ensure proper governance.

Q6) Every year, a month long ecologically important campaign/festival is held during which certain communities/tribes plant samplings of fruit-bearing trees. Which of the following are such communities/tribes ?

(a) Bhutia and Lepcha

(b) Gond and Korku

(c) Irula and Toda

(d) Sahariya and Agariya6. Consider the following statements:

Statement-I:

The European Parliament approved The Net-Zero Industry Act recently.

Statement-II:

The European Union intends to achieve carbon neutrality by 2040 and therefore aims to develop all of its own clean technology by that time.

Which one of the following is correct in respect of the above statements?

(a) Both Statement-I and Statement-II are correct and Statement-II explains Statement-I

(b) Both Statement-I and Statement-II are correct, but Statement-II does not explain Statement-I

(c) Statement-I is correct, but Statement-II is incorrect

(d) Statement-I is incorrect, but Statement-II is correct

Show Answer

Ans: (a)

Explanation

Statement I is correct:

Statement-II is correct: "The European Union intends to achieve carbon neutrality by 2040 and therefore aims to develop all of its own clean technology by that time."

7. Consider the following statements:

Statement-1:

Recently, Venezuela has achieved a rapid recovery from its economic crisis and succeeded in preventing its people from fleeing/emigrating to other countries.

Statement-II:

Venezuela has the world's largest oil reserves.

Which one of the following is correct in respect of the above statements?

(a) Both Statement-I and Statement-II are correct and Statement-II explains Statement-I

(b) Both Statement-I and Statement-II are correct, but Statement-II - does not explain Statement-I

(c) Statement-I is correct, but Statement-II is incorrect

(d) Statement-I is incorrect, but Statement-II is correct

Show Answer

Ans: (d)

Explanation

Statement I is incorrect: Venezuela has been facing a severe economic crisis characterized by hyperinflation, economic contraction, shortages of basic goods, and political instability. This crisis has led to a significant emigration of people fleeing the country in search of better economic opportunities and living conditions. There has been no recent rapid recovery that would prevent emigration.

Statement is correct: Venezuela is known to have the largest proven oil reserves in the world, surpassing even Saudi Arabia's reserves. Oil has historically been a major source of revenue for Venezuela's economy

8. With reference to the Digital India Land Records Modernisation Programme, consider the following statements:

1. To implement the scheme, the Central Government provides 100% funding.

2. Under the Scheme, Cadastral Maps are digitised.

3. An initiative has been undertaken to transliterate the Records of Rights from local language to any of the languages recognized by the Constitution of India.

Which of the statements given above are correct?

(a) 1 and 2 only

(b) 2 and 3 only

(c) 1 and 3 only

(d) 1, 2 and 3

Show Answer

Ans: (d)

Explanation

Statement 1 is correct: The Digital India Land Records Modernisation Programme (DILRMP) is a scheme where the central government provides 100% funding to the states and Union Territories (UTs) for implementation. The funding covers activities such as computerization of land records, digitization of cadastral maps, survey/re-survey, computerization of registration, etc.

Statement II is correct: One of the key objectives of the DILRMP is to digitize cadastral maps. Cadastral maps are detailed maps showing the boundaries of land parcels, which are crucial for land administration and management.

Statement 3 is correct: As part of the DILRMP, efforts have been made to standardize and digitize Records of Rights (RoR) and other land records, including transliterating them from local languages to languages recognized by the Constitution of India. This facilitates easier access and understanding of land records by stakeholders.

9. With reference to the 'Pradhan Mantri Surakshit Matritva Abhiyan', consider the following statements:

1. This scheme guarantees a minimum package of antenatal care services to women in their second and third trimesters of pregnancy and six months post-delivery health care service in any government health facility.

2. Under this scheme, private sector health care providers of certain specialities can volunteer to provide services at nearby government health facilities.

Which of the statements given above is/are correct?

(a) 1 only

(b) 2 only

(c) Both 1 and 2

(d) Neither 1 nor 2

Show Answer

Ans: (b)

Explanation Statement 1 is incorrect: The Pradhan Mantri Surakshit Matritva Abhiyan (PMSMA) aims to provide free antenatal care services to pregnant women on the 9th of every month. It focuses on providing quality antenatal check-ups to pregnant women in their second and third trimesters. However, it does not guarantee six months of post-delivery health care service in any government health facility. Post-delivery care is not part of the PMSMA's primary objectives.

Statement 2 is correct: The PMSMA encourages private sector healthcare providers, especially specialists like obstetricians, gynecologists, physicians, and radiologists, to volunteer and provide free services at designated government health facilities on the 9th of every month.

10. With reference to the Pradhan Mantri Shram Yogi Maan-dhan (PM-SYM) Yojana, consider the following statements :

1. The entry age group for enrolment in the scheme is 21 to 40 years.

2. Age specific contribution shall be made by the beneficiary.

3. Each subscriber under the scheme shall receive a minimum pension of ₹ 3,000 per month after attaining the age of 60 years.

4. Family pension is applicable to the spouse and unmarried daughters.

Which of the statements given above is/are correct?

(a) 1, 3 and 4

(b) 2 and 3 only

(c) 2 only

(d) 1, 2 and 4

Show Answer

Ans: (b)

Explanation

The entry age group for enrolment in the Pradhan Mantri Shram Yogi Maan-dhan (PM-SYM) scheme is 18 to 40 years, not 21 to 40 years.

Age-specific contribution shall be made by the beneficiary under the PM-SYM scheme. The contribution amount varies based on the age of the subscriber at the time of joining the scheme.

Each subscriber under the PM-SYM scheme shall receive a minimum pension of ₹3,000 per month after attaining the age of 60 years.

Family pension is applicable only to the spouse of the subscriber, not to unmarried daughters

11. Consider the following statements regarding 'Nari Shakti Vandan Adhiniyam':

1. Provisions will come into effect from the 18th Lok Sabha.

2. This will be in force for 15 years after becoming an Act.

3. There are provisions for the reservation of seats for Scheduled Castes Women within the quota reserved for the Scheduled Castes.

Which of the statements given above are correct?

(a) 1, 2 and 3

(b) 1 and 2 only

(c) 2 and 3 only

(d) 1 and 3 only

Show Answer

Ans: (c)

Explanation

Statement 1 is incorrect: The reservation will be effective after the census conducted after the commencement of this Bill has been published. Based on the census, delimitation will be undertaken to reserve seats for women.

Statement 2 is correct: The reservation will be provided for a period of 15 years. However, it shall continue till such date as determined by a law made by Parliament.

Statement 3 is correct: The Bill introduces Article 332A, which mandates the reservation of seats for women in every state Legislative Assembly. Additionally, one-third of the seats reserved for SCs and STs must be allocated for women, and one-third of the total seats filled through direct elections to the Legislative Assemblies shall also be reserved for women

12. Which of the following statements about 'Exercise Mitra Shakti-2023' are correct?

1. This was a joint military exercise between India and Bangladesh.

2. It commenced in Aundh (Pune).

3. Joint response during counter-terrorism operations was a goal of this operation.

4. Indian Air Force was a part of this exercise.

Select the answer using the code given below:

(a) 1, 2 and 3

(b) 1, 2 and 4

(c) 1, 3 and 4

(d) 2, 3 and 4

Show Answer

Ans: (d)

Explanation

Statement1 is incorrect: This is a joint military exercise between Indian and Sri Lankan armies.

Statement 2 is correct It commenced in Pune.

Statement 3 is correct The exercise includes synergising joint responses during counter-terrorist operations.

 Statement 4 is correct The Indian air force was a part of this exercise. Fifteen personnel from the IAF participated in the exercise, along with five personnel from the Sri Lankan Air Force.


13. A Writ of Prohibition is an order issued by the Supreme Court or High Courts to:

(a) a government officer prohibiting him from taking a particular action.

(b) the Parliament/Legislative Assembly to pass a law on Prohibition.

(c) the lower court prohibiting continuation of proceedings in a case.

(d) the Government prohibiting it from following an unconstitutional policy.

Show Answer

Ans: (c)

Explanation

A Writ of Prohibition is an order issued by a higher court, such as the Supreme Court or High Courts, directing a lower court or tribunal to cease proceedings in a case that exceeds its jurisdiction or lacks jurisdiction altogether. This writ is typically used to prevent a lower court from continuing with a case that does not fall within its legal authority, ensuring that legal boundaries are respected and jurisdictional overreach is prevented.

Thus correct answer is (c)


14. Consider the following statements:

1. It is the Governor of the State who recognizes and declares any community of that State as a Scheduled Tribe.

2. A community declared as a Scheduled Tribe in a State need not be so in another State.

Which of the statements given above is/are correct?

(a) 1 only

(b) 2 only

(c) Both 1 and 2

(d) Neither 1 nor 2

Show Answer

Ans: (b)

Explanation

Statement 1 is Incorrect: The recognition and declaration of any community as a Scheduled Tribe is not within the powers of the Governor of the State. This authority rests with the President of India. Under Article 342 of the Indian Constitution, the President may, after consultation with the Governor of a state, specify the tribes or tribal communities which shall be deemed to be Scheduled Tribes in relation to that state or Union Territory.

Statement 2 is Correct: A community recognized as a Scheduled Tribe in one state may not necessarily be recognized as such in another state. The recognition of Scheduled Tribes is state-specific, and a tribe listed as a Scheduled Tribe in one state is not automatically considered a Scheduled Tribe in another state. This is because the criteria and conditions for recognition can vary across states based on regional contexts and historical factors.

15. With reference to Union Budget, consider the following statements:

1. The Union Finance Minister on behalf of the Prime Minister lays the Annual Financial Statement before both the Houses of Parliament.

2. At the Union level, no demand for a grant can be made except on the recommendation of the President of India.

Which of the statements given above is/are correct?

(a) 1 only

(b) 2 only

(c) Both 1 and 2

(d) Neither 1 nor 2

Show Answer

Ans: (b)

Explanation

Statement 1 is Incorrect: The Annual Financial Statement, commonly referred to as the Budget, is laid before both Houses of Parliament by the Union Finance Minister, but it is not specifically done on behalf of the Prime Minister. The finance minister presents the budget on behalf of the government, as outlined in Article 112 of the Indian Constitution. There is no formal requirement for the Prime Minister's name to be associated with this act.

Statement 2 is Correct: According to Article 113(3) of the Indian Constitution, no demand for a grant can be made except on the recommendation of the President of India. This ensures that financial proposals are under the scrutiny and approval of the executive before being placed in Parliament for consideration.


16. Who of the following is the author of the books "The India Way" and "Why Bharat Matters"?

(a) Bhupender Yadav

(b) Nalin Mehta

(c) Shashi Tharoor

(d) Subrahmanyam Jaishankar

Show Answer

Ans: (d)

Explanation

The India Way" was published in 2020 and presents a perspective on India's foreign policy and strategic outlook.

"Why Bharat Matters" was published in 2021 and discusses various facets of India's importance on the global stage.

Subrahmanyam Jaishankar is an Indian diplomat and politician who has served as the External Affairs Minister of India.

17. Consider the following pairs:

Country Reason for being in the news
Argentina Worst economic crisis
Sudan War between the country's regular army and paramilitary forces
Turkey Rescinded its membership of NATO


How many of the pairs given above are correctly matched?

(a) Only one pair

(b) Only two pairs

(c) All three pairs

(d) None of the pairs

Show Answer

Ans: (b)

Explanation

Argentina has been in the news for its economic crisis, which has been ongoing for several years. The crisis is characterised by high inflation, a large public debt, and a history of economic instability.

The conflict in Sudan has escalated into a devastating war between the country’s regular military and a powerful paramilitary force, with grave consequences for the civilian population and the potential for further atrocities and humanitarian crises.

Turkey has not rescinded its membership to NATO. However, it has been in the news recently due to its opposition to Sweden’s bid to join the alliance

18. Consider the following statements:

Statement-I:

Sumed pipeline is a strategic route for Persian Gulf oil and natural gas shipments to Europe.

Statement-II:

Sumed pipeline connects the Red Sea with the Mediterranean Sea.

Which one of the following is correct in respect of the above statements?

(a) Both Statement-I and Statement-II are correct and Statement-II explains Statement-I

(b) Both Statement-I and Statement-II are correct, but Statement-II does not explain Statement-I

(c) Statement-I is correct, but Statement-II is incorrect

(d) Statement-I is incorrect, but Statement-II is correct

Show Answer

Ans: (b)

Explanation

Statement-I is correct: The Sumed pipeline, also known as the Suez-Mediterranean pipeline, is indeed a strategic route for transporting oil from the Persian Gulf (via the Red Sea) to the Mediterranean Sea. It provides an alternative route for oil shipments to Europe, bypassing the Suez Canal.

Statement-II is correct: This statement correctly describes the Sumed pipeline, which runs from the Ain Sukhna terminal on the Red Sea coast of Egypt to the Sidi Kerir terminal on the Mediterranean coast of Egypt.

Statement-3 correctly describes the Sumed pipeline's role in transporting Persian Gulf oil and natural gas to Europe. Statement-II accurately describes the geographical route of the Sumed pipeline, connecting the Red Sea (via the Gulf of Suez) with the Mediterranean Sea. However, Statement-II does not provide an explanation for Statement-I; instead, both statements independently describe different aspects of the Sumed pipeline.

19. Consider the following statements :

1. The Red Sea receives very little precipitation in any form.

2. No water enters the Red Sea from rivers.

Which of the statements given above is/are correct?

(a) 1 only

(b) 2 only

(c) Both 1 and 2

(d) Neither 1 nor 2

Show Answer

Ans: (c)

Explanation

Statement 1 is correct. The Red Sea region is characterized by arid and semi-arid climates, with extremely low precipitation levels. The area surrounding the Red Sea, including Egypt, Saudi Arabia, Sudan, and Eritrea, generally receives minimal rainfall throughout the year.

Statement 2 is correct: The Red Sea is mostly isolated from major river systems. the amount of freshwater entering the Red Sea from rivers is negligible compared to other seas. The Red Sea's primary source of water is through seawater inflow.

20. According to the Environmental Protection Agency (EPA), which one of the following is the largest source of sulphur dioxide emissions?

(a) Locomotives using fossil fuels

(b) Ships using fossil fuels

(c) Extraction of metals from ores

(d) Power plants using fossil fuels

Show Answer

Ans: (d)

Explanation

According to Environmental Protection Agency (EPA) , Emissions that lead to high concentrations of SO2 generally also lead to the formation of other SOx. The largest sources of SO2 emissions are from fossil fuel combustion at power plants and other industrial facilities.

Q21) The 1929 Session of Indian National Congress is on significance in the history of the Freedom Movement because the

(a) attainment of Self-Government was declared as the objective of the Congress.

(b) Attainment of Poorna Swaraj was adopted as the goal of the Congress.

(c) Non-Cooperation Movement was launched.

(d) Decision to participate in the Round Table Conference in London was taken.21. How many Delimitation Commissions have been constituted by the Government of India till December 2023?

(a) One

(b) Two

(c) Three

(d) Four

Show Answer

Ans: (d)

Explanation

Four delimitation commissions

First Delimitation Commission: Constituted in 1952 under the Delimitation Commission Act, 1952.

Second Delimitation Commission: Constituted in 1963 under the Delimitation Commission Act, 1962.

Third Delimitation Commission: Constituted in 1973 under the Delimitation Act, 1972.

Fourth Delimitation Commission: Constituted in 2002 under the Delimitation Act, 2002.

These commissions are responsible for the periodic readjustment of electoral constituencies for the Lok Sabha and state legislative assemblies based on changes in population distribution

22. The Constitution (71st Amendment) Act, 1992 amends the Eighth Schedule to the Constitution to include which of the following languages?

1. Konkani

2. Manipuri

3. Nepali

4. Maithili

Select the correct answer using the code given below:

(a) 1, 2 and 3

(b) 1, 2 and 4

(c) 1, 3 and 4

(d) 2, 3 and 4

Show Answer

Ans: (a)

Explanation

The Eighth Schedule to the Constitution of India was amended in accordance with the 71st amendment of the Indian Constitution. The inclusion of the following languages was the primary reason behind the enforcement of the 71st amendment of the Indian Constitution: Konkani, Nepali and Meitei (Manipuri).

Thus correct answer is (a)

23. Which of the following statements are correct about the Constitution of India?

1. Powers of the Municipalities are given in Part IX A of the Constitution.

2. Emergency provisions are given in Part XVIII of the Constitution.

3. Provisions related to the amendment of the Constitution are given in Part XX of the Constitution.

Select the answer using the code given below:

(a) 1 and 2 only

(b) 2 and 3 only

(c) 1 and 3 only

(d) 1, 2 and 3

Show Answer

Ans: (b)

Explanation

Only two pairs are correctly matched

 Dr. Shyama Prasad Mukherjee was associated with the Bharatiya Jana Sangh, and Jagjivan Ram was associated with Congress for Democracy.

However, C. Rajagopalachari was associated with the Swatantra Party, not the Socialist Party, and Acharya Narendra Dev was associated with the Socialist Party, not Swatantra Party

24. Consider the following pairs:

S.No Party Leader
1 Bharatiya Jana Sangh Dr. Shyama Prasad Mukherjee
2 Socialist Party C. Rajagopalachari
3 Congress for Democracy Jagjivan Ram
4 Swatantra Party Acharya Narendra Dev



How many of the above are correctly matched?

(a) Only one

(b) Only two

(c) Only three

(d) All four

Show Answer

Ans: (d)

Explanation

Statement 1 is Correct: Part IX A of the Constitution of India deals with the municipalities (Urban Local Bodies). It was inserted by the 74th Amendment Act, 1992, and outlines the powers, composition, and responsibilities of urban local bodies.

Statement 2 is Correct: Emergency provisions are actually given in Part XVIII of the Constitution. This part deals with the provisions related to the Emergency (Article 352 to 360).

Statement 3 is Correct: Part XX of the Constitution deals with the amendment procedure of the Constitution (Article 368).

Thus correct answer is (d)

25. Which one of the following statements is correct as per the Constitution of India?

(a) Inter-State trade and commerce is a State subject under the State List.

(b) Inter-State migration is a State subject under the State List.

(c) Inter-State quarantine is a Union subject under the Union List.

(d) Corporation tax is a State subject under the State List.

Show Answer

Ans: (c)

Explanation

Inter-State quarantine is a Union subject under the Union List.(entry number 81)


26. Under which of the following Articles of the Constitution of India, has the Supreme Court of India placed the Right to Privacy?

(a) Article 15

(b) Article 16

(c) Article 19

(d) Article 21

Show Answer

Ans: (c)

Explanation

Inter-State quarantine is a Union subject under the Union List.(entry number 81)


27. What are the duties of the Chief of Defence Staff (CDS) as Head of the Department of Military Affairs?

1. Permanent Chairman of Chiefs of Staff Committee

2. Exercise military command over the three Service Chiefs

3. Principal Military Advisor to Defence Minister on all tri-service matters

Select the correct answer using the code given below :

(a) 1, 2 and 3

(b) 1 and 2 only

(c) 2 and 3 only

(d) 1 and 3 only

Show Answer

Ans: (d)

Explanation

Statement 1 is correct: The CDS acts as the Permanent Chairman of the Chiefs of Staff Committee. The COSC includes the three Service Chiefs (Army, Navy, Air Force) and the CDS. The CDS chairs this committee, which facilitates joint manship and integration among the armed forces.

Statement 2 is incorrect: The CDS does not exercise direct military command over the three Service Chiefs. Each Service Chief retains operational command over their respective services. The role of the CDS is more about integration, joint planning, and providing strategic military advice rather than operational command.

Statement 3 is correct: The CDS serves as the principal military advisor to the Defence Minister on all tri-service matters. This includes advising on matters related to joint military operations, procurement, training, and other areas where integration and jointness among the services are crucial.

28. Operations undertaken by the Army towards upliftment of the local population in remote areas to include addressing of their basic needs is called:

(a) Operation Sankalp

(b) Operation Maitri

(c) Operation Sadbhavana

(d) Operation Madad

Show Answer

Ans: (c)

Explanation

Operation Sadbhavana is a term used by the Indian Army to describe its efforts towards winning the hearts and minds of the local population in conflict-affected or remote areas. It focuses on humanitarian assistance, developmental activities, and fostering goodwill among the local communities.

The objective of Operation Sadbhavana is to create a positive environment by addressing the basic needs of the local population such as healthcare, education, infrastructure development, and socio-economic upliftment.

29. The longest border between any two countries in the world is between:

(a) Canada and the United States of America

(b) Chile and Argentina

(c) China and India

(d) Kazakhstan and Russian Federation

Show Answer

Ans: (a)

Explanation

The border between Canada and the United States is the longest international border in the world, stretching for approximately 8,891 kilometers (5,525 miles). This border is known for its peaceful relations and cooperation between the two countries.


30. Which of the following statements about the Ethics Committee in the Lok Sabha are correct?

1. Initially it was an ad-hoc Committee.

2. Only a Member of the Lok Sabha can make a complaint relating to unethical conduct of a member of the Lok Sabha.

3. This Committee cannot take up any matter which is sub-judice.

Select the answer using the code given below:

(a) 1 and 2 only

(b) 2 and 3 only

(c) 1 and 3 only

(d) 1, 2 and 3

Show Answer

Ans: (c)

Explanation

Statement 1 is correct: In the case of Lok Sabha, a study group of the house committee of privileges in 1997 recommended the constitution of an Ethics committee, but it could not be taken up by Lok sabha.

The Committee of Privileges finally recommended the constitution of an Ethics Committee during the 13th Lok Sabha.

The late Speaker, G M C Balayogi, constituted an ad hoc Ethics Committee in 2000, which became a permanent part of the House only in 2015.

Statement 2 is incorrect: Not only Members of the Lok Sabha but also any individual can make a complaint regarding the unethical conduct of a member of the Lok Sabha to the Ethics Committee. The Ethics Committee can receive complaints from various sources, including members of the public, media reports, etc.

Statement 3 is correct: The Ethics Committee generally avoids taking up matters that are sub-judice (under judicial consideration).

31. Who was the Provisional President of the Constituent Assembly before Dr. Rajendra Prasad took over?

(a) C. Rajagopalachari

(b) Dr. B.R. Ambedkar

(c) T.T. Krishnamachari

(d) Dr. Sachchidananda Sinha

Show Answer

Ans: (d)

Explanation

                                                                                                         was the first President (pro tem) of the Constituent Assembly of India. He served as the interim or provisional President until the permanent President, Dr. Rajendra Prasad, was elected to the position.

32. With reference to the Government of India Act, 1935, consider the following statements:

1. It provided for the establishment of an All India Federation based on the union of the British Indian Provinces and Princely States.

2. Defence and Foreign Affairs were kept under the control of the federal legislature.

Which of the statements given above is/are correct?

(a) 1 only

(b) 2 only

(c) Both 1 and 2

(d) Neither 1 nor 2

Show Answer

Ans: (a)

Explanation

Statement 1 is correct: The Government of India Act, 1935, proposed the establishment of an All India Federation which would have included both the British Indian provinces and the princely states. The federation was intended to have two levels of government: the federal government at the center and provincial governments in British Indian provinces.

Statement 2 is incorrect: According to the Government of India Act, 1935, defence and foreign affairs were reserved and remained under the control of the British Governor-General and his Executive Council, which operated independently of the federal legislature

33. Which one of the following is a work attributed to playwright Bhasa?

(a) Kavyalankara

(b) Natyashastra

(c) Madhyama-vyayoga

(d) Mahabhashya

Show Answer

Ans: (c)

Explanation

Bhasa was an ancient Indian playwright who is believed to have lived sometime between the 3rd century BCE and the 3rd century CE.

"Madhyama-vyayoga" is one of the plays attributed to Bhasa. It is a Sanskrit drama that narrates the story of a love triangle involving a king, a courtesan, and a young man.

Bhasa is known for his unique style and contribution to Sanskrit drama, and "Madhyama-vyayoga" is one of the surviving plays attributed to him.

34. Sanghabhuti, an Indian Buddhist monk, who travelled to China at the end of the fourth century AD, was the author of a commentary on :

(a) Prajnaparamita Sutra

(b) Visuddhimagga

(c) Sarvastivada Vinaya

(d) Lalitavistara

Show Answer

Ans: (c)

Explanation

Sanghabhuti is historically noted for his role in transmitting the Sarvastivada Vinaya, a significant Buddhist text related to monastic discipline, to China.

The Sarvastivada Vinaya is a set of rules and regulations governing the conduct of monks and nuns in the Sarvastivada tradition of Buddhism.

Sanghabhuti's commentary on the Sarvastivada Vinaya was influential in China and contributed to the understanding and practice of monastic discipline in Chinese Buddhism.

35. Consider the following properties included in the World Heritage List released by UNESCO:

1. Shantiniketan

2. Rani-ki-Vav

3. Sacred Ensembles of the Hoysalas

4. Mahabodhi Temple Complex at Bodhgaya

How many of the above properties were included in 2023?

(a) Only one

(b) Only two

(c) Only three

(d) All four

Show Answer

Ans: (b)

Explanation

As per UNESCO, World Heritage List , Shantiniketan and the Sacred Ensembles of Hoysalas were included in the list in 2023.

Mahabodhi Temple Complex at Bodhgaya (2002) and Rani ki Vav (2014) were inscribed earlier.

36. As per Article 368 of the Constitution of India, the Parliament may amend any provision of the Constitution by way of:

1. Addition

2. Variation

3. Repeal

Select the correct answer using the code given below:

(a) 1 and 2 only

(b) 2 and 3 only

(c) 1 and 3 only

(d) 1, 2 and 3

Show Answer

Ans: (d)

Explanation

As per Article 368 of the Constitution of India, which deals with the procedure for amending the Constitution, the Parliament may amend any provision of the Constitution by way of:

Addition: Parliament can add new provisions to the Constitution.

Variation: Parliament can change or alter existing provisions of the Constitution.

Repeal: Parliament can repeal (remove) existing provisions from the Constitution.

Thus, the correct answer is (d)

37. Consider the following countries :

1. Italy

2. Japan

3. Nigeria

4. South Korea

5. South Africa

Which of the above countries are frequently mentioned in the media for their low birth rates, or ageing population or declining population?

(a) 1, 2 and 4

(b) 1, 3 and 5

(c) 2 and 4 only

(d) 3 and 5 only

Show Answer

Ans: (a)

Explanation

Italy: Italy has been experiencing low birth rates and an ageing population for several years. It is one of the countries in Europe facing demographic challenges with a declining population.

Japan: Japan has one of the lowest birth rates in the world coupled with a rapidly ageing population. The country has been grappling with demographic issues such as a shrinking workforce and increased healthcare costs due to its ageing society.

South Korea: South Korea is also facing similar demographic challenges with low birth rates and an ageing population. The government has implemented various policies to encourage childbirth and address the implications of an ageing society.

38. Which of the following statements are correct in respect of a Money Bill in the Parliament ?

1. Article 109 mentions special procedure in respect of Money Bills.

2. A Money Bill shall not be introduced in the Council of States.

3. The Rajya Sabha can either approve the Bill or suggest changes but cannot reject it.

4. Amendments to a Money Bill suggested by the Rajya Sabha have to be accepted by the Lok Sabha.

Select the answer using the code given below:

(a) 1 and 2 only

(b) 2 and 3 only

(c) 1, 2 and 3

(d) 1, 3 and 4

Show Answer

Ans: (c)

Explanation

Statement 1 is correct:

Statement 2 is correct: Article 110(1) specifies that a Money Bill can only be introduced in the Lok Sabha (House of the People) and not in the Rajya Sabha (Council of States).

Statement 3 is correct: When a Money Bill is passed by the Lok Sabha, it is transmitted to the Rajya Sabha for its recommendations. The Rajya Sabha must return the Bill within 14 days, either with its recommendations or without any recommendations. The Rajya Sabha cannot reject a Money Bill. It does not have the power to amend the Bill in a manner that includes provisions other than those related to matters specified in Article 110(1). The Lok Sabha may or may not accept the recommendations of the Rajya Sabha.

Statement 4 is incorrect: The Lok Sabha is not bound to accept any amendments suggested by the Rajya Sabha to a Money Bill. It has the discretion to accept, reject, or modify these amendments.

39. Which of the following is/are correctly matched in terms of equivalent rank in the three services of Indian Defence forces?

S.No Army Airforce Navy
1 Brigadier Air Commodore Commander
2 Major General Air Vice Marshal Vice Admiral
3 Major Squadron Leader Lieutenant Commander
4 Lieutenant Colonel Group Captain Captain

Select the correct answer using the code given below :

(a) 1 and 4

(b) 1 and 3

(c) 2, 3 and 4

(d) 3 only

Show Answer

Ans: (d)

Explanation

Only 3 row is correctly matched

The ranks in the Indian Armed Forces across the Army, Air Force, and Navy are equivalent as follows:

Brigadier - Air Commodore - Commodore

Brigadier in the Army is equivalent to Air Commodore in the Air Force and Commodore in the Navy.

Major General - Air Vice Marshal - Rear Admiral

Major General in the Army is equivalent to Air Vice Marshal in the Air Force and Rear Admiral in the Navy.

Major - Squadron Leader - Lieutenant Commander

Major in the Army is equivalent to Squadron Leader in the Air Force and Lieutenant Commander in the Navy.

Lieutenant Colonel - Group Captain - Captain

Lieutenant Colonel in the Army is equivalent to Group Captain in the Air Force and Captain in the Navy.

40. The North Eastern Council (NEC) was established by the North Eastern Council Act, 1971. Subsequent to the amendment of NEC Act in 2002, the Council comprises which of the following members?

1. Governor of the Constituent State

2. Chief Minister of the Constituent State

3. Three Members to be nominated by the President of India

4. The Home Minister of India

Select the correct answer using the code given below:

(a) 1, 2 and 3 only

(b) 1, 3 and 4 only

(c) 2 and 4 only

(d) 1, 2, 3 and 4

Show Answer

Ans: (d)

Explanation

North Eastern Council (NEC) is under the administrative purview of the Ministry of Development of the North-Eastern Region (MDoNER).

NEC is not a constitutional body, but a statutory organization established under the North Eastern Council Act, 1971, as amended in 2002.

It is the nodal agency for the economic and social development of the North Eastern Region which consists of the eight States of Arunachal Pradesh, Assam, Manipur, Meghalaya, Mizoram, Nagaland, Sikkim and Tripura.

The Council comprises Governors and Chief Ministers of constituent States and three members nominated by the President.

Thus correct answer is (d)

41. Consider the following statements :

Statement-1:

If the United States of America (USA) were to default on its debt, holders of US Treasury Bonds will not be able to exercise their claims to receive payment.

Statement-II:

The USA Government debt is not backed by any hard assets, but only by the faith of the Government.

Which one of the following is correct in respect of the above statements ?

(a) Both Statement-I and Statement-II are correct and Statement-II explains Statement-I

(b) Both Statement-I and Statement-II are correct, but Statement-II does not explain Statement-I

(c) Statement-I is correct, but Statement-II is incorrect

(d) Statement-I is incorrect, but Statement-II is correct

Show Answer

Ans: d

Explanation

This statement 1 is incorrect: The US has never defaulted on its debt obligations, and US Treasury Bonds are considered one of the safest investments globally precisely because they are backed by the full faith and credit of the US government. In the event of any financial distress, the US government has always prioritized debt repayment, making default extremely unlikely. Therefore, holders of US Treasury Bonds have historically received their payments as scheduled.

Statement 2 is correct: US Government debt, including Treasury Bonds, is not backed by specific assets such as gold or land. Instead, it is backed by the creditworthiness of the US government itself. Investors trust that the US government will honor its debt obligations because of its ability to collect taxes, print money, and maintain a stable economy. This trust in the government's ability and willingness to repay its debt is what supports the value of US Treasury securities.

42. Consider the following statements:

Statement-I:

Syndicated lending spreads the risk of borrower default across multiple lenders.

Statement-II:

The syndicated loan can be a fixed amount/lump sum of funds, but cannot be a credit line.

Which one of the following is correct in respect of the above statements?

(a) Both Statement-I and Statement-II are correct and Statement-II explains Statement-I

(b) Both Statement-I and Statement-II are correct, but Statement-II does not explain Statement-I

(c) Statement-I is correct, but Statement-II is incorrect

(d) Statement-I is incorrect, but Statement-II is correct

Show Answer

Ans: (c)

Explanation

Statement I is correct: Syndicated lending involves multiple banks or financial institutions coming together to provide funds to a borrower. By spreading the loan amount across several lenders, the risk of default by the borrower is diversified among them. Each lender takes a portion of the total loan, thereby reducing their individual exposure to the borrower's credit risk.

Statement II is incorrect: Syndicated loans can indeed be structured as either a fixed amount/lump sum of funds or as a credit line (revolving credit facility). In a fixed amount syndicated loan, the borrower receives a one-time lump sum amount which they repay over a specified period. In a credit line syndicated loan, the borrower has access to a predetermined credit limit and can draw funds as needed, up to the limit, and repay based on their usage.

43. Consider the following statements in respect of the digital rupee:

1. It is a sovereign currency issued by the Reserve Bank of India (RBI) in alignment with its monetary policy.

2. It appears as a liability on the RBI's balance sheet.

3. It is insured against inflation by its very design.

4. It is freely convertible against commercial bank money and cash.

Which of the statements given above are correct?

(a) 1 and 2 only

(b) 1 and 3 only

(c) 2 and 4 only

(d) 1, 2 and 4

Show Answer

Ans: (d)

Explanation

Statement 1 is correct. The digital rupee is a digital version of the Indian rupee issued by the Reserve Bank of India (RBI), which is the central bank of India. As a sovereign currency, it is issued and regulated by the RBI in line with its monetary policy objectives.

Statement 2 is correct. Just like physical currency (cash), the digital rupee issued by the RBI would appear as a liability on its balance sheet. This is because the RBI is obligated to redeem the digital rupees for cash or commercial bank money upon demand

Statement 3 is incorrect. The design of the digital rupee does not inherently provide insurance against inflation. Inflation is managed through the RBI's monetary policy tools, including interest rates, open market operations, reserve requirements, etc. The digital rupee, like physical currency, is subject to the effects of inflation based on broader economic factors.

Statement 4 is correct. The digital rupee, like physical currency, would be convertible against commercial bank money (digital deposits) and cash. Holders of digital rupees should be able to exchange them for other forms of money (bank deposits or cash) at par value.

44. With reference to ancient India, Gautama Buddha was generally known by which of the following epithets ?

1. Nayaputta

2. Shakyamuni

3. Tathagata

Select the correct answer using the code given below:

(a) 1 only

(b) 2 and 3 only

(c) 1, 2 and 3

(d) None of the above are epithets of Gautama Buddha

Show Answer

Ans: (b)

Explanation

Shakyamuni: This epithet means "Sage of the Shakya clan." It emphasizes his birth in the Shakya clan as Siddhartha Gautama before he attained enlightenment.

Tathagata: This term is used in Buddhist literature to refer to Gautama Buddha. It means "One who has thus come" or "One who has thus gone." It denotes his realization of the ultimate truth and his attainment of enlightenment.

Option (a) is incorrect because epithet Nayaputta is associated with Mahavir

45. Consider the following statements:

S.No Archaeological Site State Description
1 Chandraketugarh Odisha Trading Port town
2 Inamgaon Maharashtra Chalcolithic site
3 Mangadu Kerala Megalithic site
4 Salihundam Andhra Pradesh Rock-cut cave shrines

In which of the above rows is the given information correctly matched?

(a) 1 and 2

(b) 2 and 3

(c) 3 and 4

(d) 1 and 4

Show Answer

Ans: (b)

Explanation

Row I is incorrectly matched. Chandraketugarh is a site in eastern India which is famous for Terracotta seals with Brahmi inscriptions.

Row II is correctly matched. Inamgaon is a famous chalcolithic site located in the state of Maharashtra famous for being known as part of Jorwe culture.

Row III is correctly matched. Mangadu is a megalithic site located in the state of Kerala.

Salihundam is located in the state of Andhra Pradesh. It is famous for historical monasteries.

46. Who of the following rulers of medieval India gave permission to the Portuguese to build a fort at Bhatkal ?

(a) Krishnadevaraya

(b) Narasimha Saluva

(c) Muhammad Shah III

(d) Yusuf Adil Shah

Show Answer

Ans: (a)

Explanation

Krishnadevaraya was a prominent ruler of the Vijayanagara Empire during the 16th century (reigned from 1509 to 1529).

He was known for his military campaigns and patronage of arts and literature.

During his reign, the Portuguese sought permission to establish a fort at Bhatkal, which was strategically located for trade and naval activities along the western coast of India.

Krishnadevaraya granted permission to the Portuguese to build a fort at Bhatkal, thereby allowing them to strengthen their presence in the region and engage in trade activities.

47. With reference to revenue collection by Cornwallis, consider the following statements:

1. Under the Ryotwari Settlement of revenue collection, the peasants were exempted from revenue payment in case of bad harvests or natural calamities.

2. Under the Permanent Settlement in Bengal, if the Zamindar failed to pay his revenues to the state, on or before the fixed date, he would be removed from his Zamindari.

Which of the statements given above is/are correct?

(a) 1 only

(b) 2 only

(c) Both 1 and 2

(d) Neither 1 nor 2

Show Answer

Ans: (b)

Explanation

Statement 1 is incorrect. The Ryotwari Settlement was introduced by Thomas Munro and not by Cornwallis. Under Ryotwari system, individual peasants (ryots) were directly responsible for paying revenue to the British government. There was no provision for exemption from revenue payment in case of bad harvests or natural calamities. The ryots bore the full burden of revenue payment regardless of crop outcomes.

Statement 2 is correct. Permanent settlement which was introduced by lord Cornwallis was made with Zamindars of Bengal. One of the negative features of this settlement was the sunset clause, under which if the Zamindar fails to pay the land revenue by the end of the day, his Zamindari would be auctioned.

48. Consider the following statements:

1. There are no parables in Upanishads.

2. Upanishads were composed earlier than the Puranas.

Which of the statements given above is/are correct?

(a) 1 only

(b) 2 only

(c) Both 1 and 2

(d) Neither 1 nor 2

Show Answer

Ans: (b)

Explanation

Statement 1 is incorrect: The term ‘parables’ refers to short stories. Upanishads is a part of Vedic texts dealing with metaphysical matters that are philosophical ideas dealing with God and soul (parmaatma and aatma respectively)

Statement 2 is correct: The Upanishads are ancient texts that are considered part of the Vedas and are among the earliest philosophical and spiritual texts in Hinduism. They were composed between 800 BCE to 400 BCE, making them older than the Puranas, which were composed later, generally from around 3rd century BCE to 16th century CE.

49. Consider the following statements :

1. India is a member of the International Grains Council.

2. A country needs to be a member of the International Grains Council for exporting or importing rice and wheat.

Which of the statements given above is/are correct?

(a) 1 only

(b) 2 only

(c) Both 1 and 2

(d) Neither 1 nor 2

Show Answer

Ans: (a)

Explanation

Statement 1 is correct: India is indeed a member of the International Grains Council (IGC). The IGC is an intergovernmental organization that deals with grains, including wheat and rice, among its member countries.

Statement 2 is incorrect: Membership in the International Grains Council is not a requirement for countries to export or import rice and wheat. The IGC facilitates international cooperation and information exchange on grains but does not have regulatory authority over trade in grains among countries.

50. Which one of the following was the latest inclusion in the Intangible Cultural Heritage List of UNESCO ?

(a) Chhau dance

(b) Durga puja

(c) Garba dance

(d) Kumbh mela

Show Answer

Ans: (c)

Explanation

Gujarat’s traditional dance form ‘Garba’ has been included in the list of Intangible Cultural Heritage by the United Nations Educational, Scientific and Cultural Organization (UNESCO).

The popular dance form is the 15th cultural item from India to make it to the UNESCO list. Kolkata’s Durga Puja was the last one added two years ago.

51. The total fertility rate in an economy is defined as :

(a) the number Of children per 1000 people in the population in a year.

(b) the number of children born to a couple in their lifetime in a given population.

(c) the birth rate minus death rate.

(d) the average number of live births a woman would have by the end of her child-bearing age.

Show Answer

Ans: (d)

Explanation

Total fertility rate (TFR) represents the average number of live births that a woman would have during her reproductive years, usually considered to be ages 15-49. It is a key demographic indicator used to understand fertility patterns within a population.

52. Consider the following Statements :

1. In India, Non-Banking Financial Companies can access the Liquidity Adjustment Facility window Of the Reserve Bank Of India.

2. In India, Foreign Institutional Investors can hold the Government Securities (G-Secs).

3. In India,  Stock Exchanges can offer separate trading platforms for debts.

Which of the statements given above is/are correct?

(a) 1 and 2 only

(b) 3 only

(c) 1,2 and 3

(d) 2 and 3 only

Show Answer

Ans: (d)

Explanation

Statement 1 is incorrect: The Liquidity Adjustment Facility (LAF) window is primarily accessed by scheduled commercial banks (SCBs) in India. NBFCs do not have direct access to the LAF window. They typically manage their liquidity through other mechanisms such as market borrowings, commercial paper issuances, etc.

Statement 2 is correct: Foreign Institutional Investors (FIIs), now known as Foreign Portfolio Investors (FPIs), are permitted to invest in Government Securities (G-Secs) under the regulatory framework set by the Reserve Bank of India (RBI) .They play a significant role in the Indian debt market.

Statement 3 is correct: In India, exchanges such as the National Stock Exchange (NSE) and Bombay Stock Exchange (BSE) offer separate trading platforms for debt instruments. These platforms allow trading in various debt securities, including corporate bonds, government securities, and other debt instruments.

53. In India, which of the following can trade in Corporate Bonds and Government Securities ?

I. Insurance Companies

2. Pension Funds

3. Retail Investors

Select the correct answer using the code given below:

(a) 1 and 2 only

(b) 2 and 3 only

(c) 1 and 3 only

(d) 1, 2 and 3

Show Answer

Ans: (d)

Explanation

Statement 1 is correct: Insurance companies in India are permitted to invest in both Corporate Bonds and Government Securities. They play a significant role in the debt market as institutional investors, contributing to the overall liquidity and stability of the market.

Statement 2 is correct: Pension funds in India, such as the Employees' Provident Fund Organization (EPFO) and the National Pension System (NPS), are also allowed to invest in Corporate Bonds and Government Securities. These investments are crucial for pension funds to generate returns while maintaining the safety and liquidity required for long-term obligations to retirees.

Statement 3 is correct: Retail investors, which include individual investors, are permitted to invest in Corporate Bonds through the retail issuance channels offered by companies or through mutual funds that invest in bonds. Government Securities (G-Secs) are also accessible to retail investors through various channels, including direct participation in government bond auctions, exchange-traded funds (ETFs), and mutual funds specializing in government bonds.

54. Consider the following :

1. Exchange-Traded Funds (ETF)

2. Motor vehicles

3. Currency swap

Which Of the above Ware considered financial instruments ?

(a) 1 only

(b) 2 and 3 only

(c) 1, 2 and 3

(d) 1 and 3 only

Show Answer

Ans: (d)

Explanation

Statement 1 is correct: ETFs are investment funds that are traded on stock exchanges, much like stocks. They hold assets such as stocks, commodities, or bonds and generally operate with an arbitrage mechanism designed to keep the trading close to its net asset value (NAV). ETFs are considered financial instruments because they represent a form of investment vehicle traded in financial markets.

Statement 2 is incorrect: Motor vehicles, such as cars and trucks, are not typically considered financial instruments. They are physical assets used primarily for transportation purposes rather than for financial investment or trading.

Statement3 is correct: A currency swap is a financial derivative contract that involves the exchange of interest and sometimes principal in one currency for the same in another currency. Currency swaps are used to hedge against foreign exchange rate fluctuations or to obtain cheaper debt in foreign currencies. They are widely used in financial markets and are considered financial instruments.

55 With reference to the sectors of the Indian economy consider the following pairs :

S.No Economic Activity Sector
1 Storage of agricultural produce Secondary
2 Dairy farm Primary
3 Mineral exploration Tertiary
4 Weaving cloth Secondary

How many of the pairs given above are correctly matched ?

(a) Only one

(b) Only two

(c) Only three

(d) All four


Show Answer

Ans: (b)

Explanation

Storage of agricultural produce: This activity is associated with the Tertiary sector, which involves storage and distribution services. Therefore, Pair 1 is not matched correctly.

Dairy farm: Dairy farming involves the primary production of milk and other dairy products from livestock. Hence, it belongs to the Primary sector. Pair 2 is matched correctly.

Mineral exploration: Mineral exploration is part of the process of finding commercially viable mineral deposits and is a part of the Primary sector, which involves extraction and production of natural resources. Pair 3 is matched incorrectly.

Weaving cloth:Weaving cloth involves the manufacturing of textile products and is a part of the Secondary sector, which includes industrial production. Pair 4 is matched correctly.

56. Consider the following materials :

1. Agricultural residues

2. Corn grain

3. Wastewater treatment sludge

4.  Woodmill waste

Which of the above can used as feedstock for producing Sustainable Aviation Fuel?

(a) 1 and 2 only

(b) 3 and 4 only

(c) 1, 2, 3 and 4

(d) 1, 3 and 4 only

Show Answer

Ans: (c)

Explanation

The production of Sustainable Aviation Fuel (SAF) focuses on utilizing feedstocks that are renewable and sustainable, thereby reducing greenhouse gas emissions compared to conventional fossil fuels.

Agricultural residues: These are leftover materials from agricultural activities, such as stalks, husks, and straw. They can be converted into biofuels through processes like biochemical conversion (e.g., fer mentation) or thermochemical conversion (e.g., pyrolysis).

Corn grain: Corn grain can be used to produce bioethanol, which is a type of biofuel. However, for aviation fuel, corn grain can be processed into biobutanol or can undergo further conversion processes to potentially contribute to SAF production.

Wastewater treatment sludge: This refers to the solid residual material generated during wastewater treatment processes. It contains organic matter that can be converted into biogas through anaerobic digestion. Biogas can then be processed into biofuels, including SAF.

Mill waste: This can include various types of waste from industrial processes, such as wood waste, pulp waste, or other by-products from milling operations. These can often be used as feedstocks for biofuel production.

57. With reference to physical capital in Indian economy, consider the following pairs:

S.No Item Category
1 Farmer's plough Working capital
2 Computer Fixed capital
3 Yarn used by the weaver Fixed capital
4 Petrol Working capital

How many of the above pairs are correctly matched ?

(a) Only one

(b) Only two

(c) Only three

(d) All four

Show Answer

Ans: (a)

Explanation

Farmer's plough: A farmer's plough is used in agricultural activities and is typically considered a part of Fixed capital. Fixed capital refers to durable goods or means of production that are used repeatedly in production processes over extended periods.

Computer: A computer is used for various purposes, including in businesses and industries, and is a classic example of Fixed capital. It is a durable asset used for performing tasks efficiently over an extended period.

Yarn used by the weaver: Yarn used by a weaver is considered a part of Working capital. Working capital includes goods that are consumed or transformed in the production process and are not retained in the same form as part of the fixed assets.

Petrol : Floating capital: Capital goods which are capable of having some alternative uses are called floating capital. For e.g. electricity, fuel, transport vehicles, etc are the floating capital which can be used anywhere.


58. Which one of the following words/phrases is most appropriately used to denote "an

interoperable network of 3D virtual worlds that can be accessed simultaneously by

millions of users, Who can exert property rights over virtual items' ?

(a) Big data analytics

(b) Cryptography

(c) Metaverse

(d) Virtual matrix

Show Answer

Ans: (c)

Explanation

Metaverse: The term "metaverse" refers to a collective virtual shared space, created by the convergence of virtually enhanced physical reality and physically persistent virtual reality. In the metaverse, users can interact with each other and digital objects, often using avatars. It is envisioned as an interconnected network of virtual worlds, allowing for seamless interaction and collaboration across different platforms and environments.

59. With reference to the rule/rules imposed by the Bank Of India while treating

foreign banks, consider the following statements :

1. There is no minimum capital requirement for wholly owned banking subsidiaries in India.

2. For wholly owned banking subsidiaries in India, at least 50% of the board members should be Indian nationals.

Which Of the statements given Ware correct ?

(a) 1 only

(b) 2 only

(c) Both 1 and 2

(d) Neither 1 nor 2

Show Answer

Ans: (b)

Explanation

Statement 1 is incorrect: The RBI imposes minimum capital requirements for foreign banks establishing wholly owned subsidiaries in India. These requirements are designed to ensure that the subsidiaries have adequate capital to support their operations and manage risks effectively in the Indian banking environment.

Statement 2 is correct: The RBI mandates that at least 50% of the board of directors of wholly owned banking subsidiaries in India must consist of Indian nationals. This requirement is aimed at ensuring local representation and governance in the operations of these subsidiaries.

This is main Question 60

60. With reference to Corporate Social Responsibility (CSR) rules in India, consider

the following statements :

1. CSR rules specify that expenditures that the directly or its employees will not be considered as CSR

activities.

1. CSR rules specify that expenditures that benefit the company the directly or its employees will not be considered as CSR

activities.

Which of the statements given above is/are correct ?

(a) 1 only

(b) 2 only

(c) Both I and 2

(d) Neither I nor 2

Show Answer

Ans: (a)

Explanation

Statement 1 is correct: As per the Companies Act, 2013, and the CSR Rules, expenditures that benefit only the company directly or its employees, such as employee welfare expenses, are explicitly excluded from being considered as CSR activities. CSR activities should primarily benefit society at large and not just the company or its employees.

Statement 2 is incorrect: CSR rules in India mandate that certain classes of companies meeting specific criteria must spend at least 2% of their average net profits made during the three immediately preceding financial years on CSR activities. This spending requirement is mandatory for eligible companies falling under the purview of the Companies Act, 2013

61. With reference to radioisotope thermoelectric generators (RTGS), consider the following statements:

1. RTGS are miniature fission reactors.

2. RTGS are used for powering the onboard systems of spacecrafts.

3. RTGS can use Plutonium-238, which is a by-product of weapons development.

Which of the statements given above are correct?

(a) 1 and 2 only

(b) 2 and 3 only

(c) 1 and 3 only

(d) 1, 2 and 3

Show Answer

Ans: (b)

Explanation

Statement I is incorrect: RTGs (Radioisotope Thermoelectric Generators) are not miniature fission reactors. Instead, they generate electricity using the heat produced by the decay of a radioactive isotope, typically Plutonium-238 or Uranium-238. They do not involve nuclear fission reactions as in traditional nuclear reactors.

Statement II is correct. RTGs have been used by space agencies like NASA to provide power to spacecraft and satellites operating in environments where solar power may not be practical or feasible, such as missions to outer planets or deep space where sunlight is weak or nonexistent.

Statement III is correct. Plutonium-238 is produced as a by-product of nuclear reactor operations, specifically in reactors designed to produce it. It has been used in RTGs because of its high heat output through radioactive decay, which can be converted into electricity through thermoelectric processes.

Prelims

More Year Papers

62. Consider the following statements:

Statement-I :

Giant stars live much longer than dwarf stars.

Statement-II:

Compared to dwarf stars, giant stars have a greater rate of nuclear reactions.

Which one of the following is correct in respect of the above statements?

(a) Both Statement-I and Statement-II are correct and Statement-II explains Statement-I

(b) Both Statement-I and Statement-II are correct, but Statement-II does not explain Statement-I

(c) Statement-I is correct, but Statement-II is incorrect

(d) Statement-I is incorrect, but Statement-II is correct

Show Answer

Ans: (d)

Explanation

Statement-I is incorrect: In reality, giant stars generally have shorter lifespans compared to dwarf stars. Giant stars are larger and more massive than dwarf stars, but they burn through their nuclear fuel at a much faster rate due to their higher rate of nuclear reactions and greater luminosity. As a result, they have shorter lifespans in astronomical terms.

Statement-II is correct: Giant stars, due to their larger size and higher mass, have stronger gravitational pressures in their cores. This leads to higher temperatures and pressures, which in turn results in faster nuclear reactions compared to dwarf stars. This increased rate of nuclear fusion contributes to the higher luminosity and shorter lifespan of giant stars.

63. Which of the following is Synthesised in human body that dilates blood vessels and increases blood flow ?

(a)Nitric Oxide

(b)Nitrous oxide

(c)Nitrogen dioxide

(d)Nitrogen Pentoxide

Show Answer

Ans: (a)

Explanation

Nitric Oxide (NO): Nitric oxide is a molecule synthesized in various cells of the human body, primarily by endothelial cells lining the blood vessels. It acts as a vasodilator, meaning it relaxes the smooth muscles of blood vessels, causing them to dilate. This dilation increases blood flow and helps regulate blood pressure. Nitric oxide also plays a crucial role in numerous physiological processes, including immune response, neurotransmission, and the regulation of cell death

64. Consider the following activities:

1. Identification of narcotics on passengers at airports or in aircraft's

2. Monitoring of Perceptions

3.Tracking the migrations of Animals

In how many of the above activities can be radars be used?

(a) Only one

(b) Only two

(c) Only three

(d) None

Show Answer

Ans: (b)

Explanation

Statement I is incorrect: Radars are not typically used for identifying narcotics. Detection of narcotics typically involves methods such as X-ray scanning, sniffer dogs, chemical analysis, and visual inspections. Radars are not suitable for detecting narcotics because they are designed for detecting objects based on reflected radio waves, which are not specific to narcotics detection.

Statement II is correct: Radars are commonly used for monitoring precipitation, such as rainfall and snowfall. Weather radars emit radio waves that bounce off precipitation particles (like raindrops or snowflakes) and return to the radar receiver. By analyzing the reflected signals, meteorologists can determine the location, intensity, and movement of precipitation in real-time.

Statement III is correct: Radars can be used to track the migrations of animals, particularly birds. Avian radar systems are designed to detect and track the movements of birds during migration. These radars use similar principles to weather radars but are optimized to detect the specific radar cross-sections of birds and distinguish them from weather phenomena.

65.Consider the following aircrafts:

1. Raefal

2.MiG-29

3.Tejas MK-1

How many of the above are consider fifth 5 generation fighter aircraft ?

(a) Only one

(b) Only two

(c) Only three

(d) None

Show Answer

Ans: (d)

Explanation

Rafale: The Dassault Rafale is a 4.5th generation multirole fighter aircraft. It is known for its versatility, advanced avionics, and weapon systems, but it does not meet all the criteria of a 5th generation fighter.

MiG-29: The Mikoyan MiG-29 is also a 4th generation fighter aircraft. It is a highly maneuverable air superiority fighter known for its agility and combat effectiveness, but it lacks the advanced stealth and sensor fusion capabilities of a 5th generation fighter.

Tejas MK-1: The HAL Tejas MK-1 is an indigenous 4th generation lightweight multirole fighter developed by India. It is designed for air superiority and ground attack missions, but similar to the Rafale and MiG-29, it does not qualify as a 5th generation fighter

66. In which of the following are hydrogels used?

1. Controlled drug delivery in patients

2. Mobile air-conditioning systems

3. Preparation of industrial lubricants

Select the correct answer using the code given below:

(a) 1 only

(b) 1 and 2 only

(c) 2 and 3 only

(d) 1, 2 and 3

Show Answer

Ans: (d)

Explanation

Statement I is correct: Hydrogels are widely used in biomedical applications, particularly for controlled drug delivery systems. Hydrogels can absorb and retain large amounts of water while maintaining their structural integrity. They can be engineered to release drugs in a controlled manner over time, providing sustained release and improving therapeutic outcomes.

Statement II is correct: Hydrogels can be used in the development of advanced materials for cooling systems, including mobile air-conditioning. These hydrogels can absorb and release water vapor in response to changes in temperature and humidity, providing efficient and sustainable cooling solutions.

Statement III is correct: Hydrogels are also used in industrial applications, including the preparation of specialized lubricants. Hydrogel-based lubricants can provide enhanced lubrication properties, reduce friction, and improve performance under various operating conditions.

67. Which one of the following is the exhaust pipe emission from Fuel Cell Electric Vehicles, powered by hydrogen?

(a) Hydrogen peroxide

(b) Hydronium

(c) Oxygen

(d) Water vapour

Show Answer

Ans: (d)

Explanation

Fuel Cell Electric Vehicles (FCEVs) use hydrogen as their fuel source and generate electricity through a chemical reaction between hydrogen and oxygen in the fuel cell stack. This reaction produces electricity to power the vehicle and the only by-product is water vapour (H2O)

Fuel Cell Electric Vehicles (FCEVs) use hydrogen as their fuel source and generate electricity through a chemical reaction between hydrogen and oxygen in the fuel cell stack. This reaction produces electricity to power the vehicle and the only by-product is water vapour (H2O)


68. Recently, the term "pumped-storage hydropower" is actually and appropriately discussed in the context of which one of the following?

(a) Irrigation of terraced crop fields

(b) Lift irrigation of cereal crops

(c) Long duration energy storage

(d) Rainwater harvesting system

Show Answer

Ans: (c)

Explanation

Pumped-storage hydropower is a method of storing energy by using two water reservoirs at different elevations. During times when electricity demand is low (off-peak), excess electricity from the grid is used to pump water from a lower reservoir to an upper reservoir. Then, during times of high electricity demand (peak), the stored water is released back down through turbines to generate electricity.


69. "Membrane Bioreactors" are often discussed in the context of:

(a) Assisted reproductive technologies

(b) Drug delivery nanotechnologies

(c) Vaccine production technologies

(d) Wastewater treatment technologies

Show Answer

Ans: (d)

Explanation

Membrane Bioreactors (MBRs) are advanced wastewater treatment technologies that combine biological treatment processes (bioreactors) with membrane filtration. In MBR systems, microorganisms biologically break down organic matter and pollutants in wastewater. The key component of MBRs is the membrane filtration system, which separates treated water from solids (biomass) in the bioreactor.

70. With reference to the Indian economy, "Collateral Borrowing and Lending Obligations" are the instruments of:

(a) Bond market

(b) Forex market

(c) Money market

(d) Stock market

Show Answer

Ans: (c)

Explanation

The money market in India facilitates short-term borrowing and lending of funds. It deals with instruments that have a maturity period of up to one year.

CBLO (Collateral Borrowing and Lending Obligations) is one such instrument traded in the money market. It allows entities to borrow or lend funds with government securities as collateral.

71. The organisms "Cicada, Froghopper and Pond skater" are:

(a) Birds

(b) Fish

(c) Insects

(d) Reptiles

Show Answer

Ans: (c)

Explanation

Cicada: Cicadas are large, often noisy insects belonging to the order Hemiptera. They are known for their distinctive songs produced by vibrating membranes on their abdomens.

Froghopper: Froghoppers, also known as spittlebugs, belong to the superfamily Cercopoidea within the order Hemiptera. They are named for their jumping ability, which is similar to that of frogs. Froghoppers are small insects that feed on plant sap and are known for the protective foam nests they create on plants as nymphs.

Pond skater: Pond skaters, or water striders, are insects belonging to the family Gerridae within the order Hemiptera. They are adapted to living on the surface of freshwater bodies such as ponds and streams.


72. Consider the following statements:

Statement-I:

Many chewing gums found in the market are considered a source of environmental pollution.

Statement-II:

Many chewing gums contain plastic as gum base.

Which one of the following is correct in respect of the above statements?

(a) Both Statement-I and Statement-II are correct and Statement-II explains Statement-I

(b) Both Statement-I and Statement-II are correct, but Statement-II does not explain Statement-I

(c) Statement-I is correct, but Statement-II is incorrect

(d) Statement-I is incorrect, but Statement-II is correct

Show Answer

Ans: (a)

Explanation

Statement-I is correct: Chewing gums, especially those that contain synthetic polymers like plastic in their gum base, do contribute to environmental pollution. When gum is discarded improperly, it can persist in the environment for a long time due to its synthetic nature, leading to littering issues in streets, parks, and other public spaces.

Statement-II is also correct: Traditional chewing gums often use synthetic polymers such as polyethylene, polyvinyl acetate, or polyisobutylene as the gum base. These polymers are not biodegradable and can remain in the environment for years after disposal, contributing to plastic pollution.

Furthermore, Statement-II logically supports Statement-I because the presence of plastic in chewing gums explains why they contribute to environmental pollution when improperly disposed of it.

73. Consider the following pairs:

S.No Country Animal found in its natural habitat
1 Brazil Indri
2 Indonesia Elk
3 Madagascar Bonobo

How many of the pairs given above are correctly matched?

(a) Only one

(b) Only two

(c) All three

(d) None

Show Answer

Ans: (d)

Explanation

Indri is a type of lemur found naturally in Madagascar, not Brazil.

Elk is found naturally in North America and parts of Eurasia, not Indonesia.

Bonobo is a species of great ape found naturally in the Democratic Republic of Congo, not Madagascar.

74. Consider the following statements regarding World Toilet Organization:

1. It is one of the agencies of the United Nations.

2. World Toilet Summit, World Toilet Day and World Toilet College are the initiatives of this organization, to inspire action to tackle the global sanitation crisis.

3. The main focus of its function is to grant funds to the least developed countries and developing countries to achieve the end of open defecation.

Which of the statements given above is/are correct?

(a) 2 only

(b) 3 only

(c) 1 and 2

(d) 2 and 3

Show Answer

Ans: (a)

Explanation

Statement 1 is incorrect. The World Toilet Organization is an NGO and not a part of the United Nations system. It is a global non-profit organization focused on improving sanitation and toilet conditions worldwide.

Statement 2 is correct. The World Toilet Organization organizes events like the World Toilet Summit and promotes observances such as World Toilet Day to raise awareness and inspire actions to address sanitation challenges globally. The World Toilet College is also an initiative aimed at improving sanitation standards through training and education.

Statement 3 is incorrect While the WTO is involved in advocacy and awareness programs to improve sanitation, it does not primarily focus on granting funds to countries for achieving specific goals like ending open defecation. Its activities are more centered around advocacy, awareness campaigns, and initiatives to improve sanitation practices globally

75. Consider the following statements:

1. Lions do not have a particular breeding season.

2. Unlike most other big cats, cheetahs do not roar.

3. Unlike male lions, male leopards do not proclaim their territory by scent marking.

Which of the statements given above are correct?

(a) 1 and 2 only

(b) 2 and 3 only

(c) 1 and 3 only

(d) 1, 2 and 3

Show Answer

Ans: (a)

Explanation

Statement 1 is correct: Lions do not have a specific breeding season; instead, they can reproduce throughout the year. Females in a pride may come into estrus at different times, and mating can occur year-round.

Statement 2 is correct. Unlike lions, tigers, leopards, and jaguars, which are known for their roaring abilities, cheetahs cannot roar. Instead, they produce a variety of vocalizations including purrs, chirps, growls, and hisses.

Statement 3 is incorrect. Male leopards do indeed mark their territories using scent markings, just like male lions. They use urine and claw scraping on trees to define and defend their territories from other males.


76. Which one of the following is the correct description of Million Farmers"?

(a) It is a platform for accelerating the transition towards and water systems that are net-zero (carbon), nature positive and that aims to increase farmer resilience.

(b) It is an international alliance and a network of individuals and farming organisations interested in supporting and strengthening the development of the organic animal husbandry.

(c) It is a digital platform fully integrated with service providers and built on blockchain that lets buyers and sellers third parties trade fertilizers and securely.

(d) It is a platform with the mission of encouraging the farmers to form Farmer  Product Organisations or Agribusiness Consortiums, thus facilitating access to global open markets to their products.

Show Answer

Ans: (a)

Explanation

100 Million Farmers is a platform supporting private and public leaders to position food and farmers as central pillars on the global climate and nature agenda, and accelerate collective action to scale climate-and nature-friendly agricultural practices.


77. Consider the following :

1. Battery storage

2. Biomass generators

3. Fuel cells

4. Rooftop solar photovoltaic units

How many of the above are considered "Distributed Energy Resources" ?

(a) Only one

(b) Only two

(c) Only three

(d) All four

Show Answer

Ans: (d)

Explanation

Battery storage: Battery storage systems are considered DERs because they can store electricity generated from various sources (such as solar or wind) and discharge it when needed. They can be deployed at various scales, from residential to utility-scale, and contribute to balancing electricity supply and demand.

Biomass generators: Biomass generators use organic materials (biomass) to produce electricity through combustion or other conversion processes. They can be decentralized and distributed across different locations, making them a form of DER that contributes to renewable energy generation.

Fuel cells: Fuel cells convert chemical energy directly into electricity through electrochemical reactions, often using hydrogen as a fuel source. They can be deployed as distributed generation units, providing on-site electricity generation with high efficiency and low emissions.

Rooftop solar photovoltaic units: Rooftop solar photovoltaic (PV) units generate electricity from sunlight using solar panels installed on rooftops of buildings or homes. They are a classic example of distributed energy resources, as they generate electricity locally and can be installed in various locations, reducing transmission losses and enhancing grid resilience.

78. Which one of the following shows a unique relationship with an insect that has coevolved

with it and that is the only insect that can pollinate this tree ?

(a) Fig

(b) Mahua

(c) Sandalwood

(d) Silk

Show Answer

Ans: (c)

Explanation

Butterflies: While most butterflies are not poisonous, there are some species that have evolved to be toxic to predators. These butterflies often sequester toxins from the plants they consume during their larval stages. Examples include butterflies from families like Papilionidae (e.g., Swallowtails) and Nymphalidae (e.g., Monarchs and Blue Morphos).

Fish: Many species of fish, especially in marine environments, can be poisonous due to the toxins they accumulate from their diet or from symbiotic relationships with toxic organisms like certain algae or bacteria. Examples include pufferfish (which contain tetrodotoxin), certain species of grouper, and some species of coral reef fish.

Frogs: Frogs are well-known for having toxic species, particularly among the family Dendrobatidae (poison dart frogs). These frogs secrete potent toxins through their skin, which serves as a defense mechanism against predators. The toxins can vary from alkaloids to peptides, and they are often brightly colored to signal their toxicity.


79. Consider the following :

1. Butterflies

2. Fish

3. Frogs

How many of the above have poisonous species among them ?

(a) Only one

(b) Only two

(c) All three

(d) None

Show Answer

Answer: (c)


80. Consider the following :

I. Cashew

2. Papaya

3. Red sanders

How many of the above trees are actually native to India ?

(a) Only one

(b) Only two

(c) All three

(d) None

Show Answer

Ans: (a)

Explanation

Cashew: Cashew (Anacardium occidentale) is native to northeastern Brazil. It was introduced to India by the Portuguese in the 16th century.

Papaya: Papaya (Carica papaya) is believed to be native to southern Mexico and neighboring Central America. It was introduced to India and other tropical regions around the world where it is now widely cultivated. Like cashew, papaya is not native to India.

Red sanders: Red sanders (Pterocarpus santalinus), also known as Red Sanderswood or Rakta Chandana, is native to the southern Eastern Ghats mountain range in India. It is known for its valuable red wood and has been historically traded and used in various cultural and medicinal practices in India.

81. Consider the following Airports :

1. Donyi Polo Airport

2. Kushinagar International Airport

3. Vijayawada International Airport

In the recent past which of the above have been constructed as Greenfield projects ?

(a)1 and 2 only

(b)2 and 3 only

(c)1 and 3 only

(d)1, 2 and 3

Show Answer

Explanation

A Greenfield project refers to the construction of new infrastructure on unused land, typically from scratch, without needing to remodel or demolish any existing structures

Donyi Polo Airport: Located in Itanagar, Arunachal Pradesh, Donyi Polo Airport was constructed as a Greenfield project..

Kushinagar International Airport: Kushinagar International Airport in Uttar Pradesh was also developed as a Greenfield project. This airport was specifically built to facilitate international pilgrimage traffic to the Buddhist pilgrimage site of Kushinagar.

Vijayawada International Airport: Vijayawada International Airport in Andhra Pradesh underwent significant expansion and upgrade, but it was not entirely a Greenfield project. It involved upgrading the existing airport infrastructure to international standards rather than starting from completely unused land.

82. With reference to "water vapour*, which of the following statements is/are correct?

1. It is a gas,' the amount of which decreases with altitude.

2. Its percentage is maximum at the poles.

Select the answer using the code given below :

(a) 1 only

(b) 2 only

(c) Both 1 and 2

(d) Neither 1 nor 2

Show Answer

Ans: (a)

Explain

Statement I is correct: Water vapor is indeed a gas in the atmosphere, and its concentration decreases with altitude. As altitude increases, the temperature generally decreases, which reduces the capacity of the air to hold water vapor. Therefore, water vapor content tends to decrease with increasing altitude.

Statement II is incorrect: Areas with warmer temperatures and higher humidity levels (such as tropical regions) can have higher concentrations of water vapor compared to colder regions like the poles. The percentage of water vapor in the atmosphere is not maximum at the poles

83. Consider the following description :

1. Annual and daily temperatures range of is low.

2.- Precipitation occurs between   throughout the year.

3. Precipitation varies between 50 cm — 250 cm.

What is this type of climate?

(a) Equatorial climate

(b) China type  climate

(c) Humid subtropical climate

(d) Marine West coast climate

Show Answer

Ans: (d)

Explanation

Marine west coast climate

The temperature of marine west coast ecosystems is very mild and does not fluctuate greatly. There are only two seasons, summer and winter. The average low is 30 degrees Fahrenheit in winter months and can reach 72 degrees F in summer months.

This region is so humid because it can receive 50-250 cm of rain in a year. Example -Portland, Oregon: it's known for being wet and generally very mild.

However, it can get rain 150 of 365 days a year, common in marine west coast climates. This region is known for its wet, humid air.

84. With reference to "Coriolis force", which of the following statements is/are correct ?

1. It increases with increase in wind velocity.

2. It is maximum at the poles and is absent at the equator.

Select the answer using the code given below :

(a) 1 only

(b) 2 only

(c) Both 1 and 2

(d) Neither 1 nor 2

Show Answer

Ans: (c)

Explanation

Statement 1 is correct: The Coriolis force does indeed increase with an increase in wind velocity.

Statement 2 is correct: The Coriolis force is strongest at the poles and decreases to zero at the equator. At the poles, where the rotational speed of the Earth is relatively small, the Coriolis force causes a significant deflection of moving objects. Near the equator, where the rotational speed of the Earth is highest, the Coriolis force becomes negligible and effectively absent

85. On June 21 every year, which of the following latitude(s) experience(s) a sunlight of more than 12 hours ?

1. Equator

2. Tropic of Cancer

3. Tropic of Capricorn

4. Arctic Circle

Select the correct answer using the code given below :

(a) 1 only

(b) 2 only

(c) 3 and 4

(d) 2 and 4

Show Answer

Ans: (d)

Explanation

On June 21st, the Tropic of Cancer experiences more than 12 hours of sunlight. This is because the Sun's rays are directly overhead at noon at this latitude during the summer solstice in the Northern Hemisphere.

On June 21st, the Arctic Circle experiences more than 12 hours of sunlight, often referred to as a period of midnight sun. This is because the Sun does not set below the horizon, resulting in continuous daylight for 24 hours or more depending on how far north you are from the Arctic Circle.

Thus , correct answer is (d)


86. One Of the following regions has the world's largest tropical peatland, which holds about three years worth of global carbon emissions from fossil fuels; and the possible destruction of which can exert detrimental effect on the global climate. Which one of the following denotes that region ?

(a) Amazon Basin

(b) Congo Basin

(c) Kikori Basin

(d) Rio de Ia Plata Basin

Show Answer

Ans: (c)

Explanation

The Congo Basin, located in Central Africa, contains vast tropical peatlands, especially in countries like the Democratic Republic of the Congo (DRC) and Republic of the Congo.

 The Congo Basin's peatlands are estimated to hold about three years' worth of global carbon emissions from fossil fuels, highlighting their critical role in global climate regulation


87. With reference to perfluoroalkyl and perfluoroalkyl substances (PFAS) that are

used in making many consumer products, consider the following statements

1. PFAS are found to widespread in drinking water, food and food packaging materials.

2. PFAS are not easily degraded in the environment.

3. Persistent exposure to PFAS can lead to bioaccumulation in animal bodies.

Which of the Statements given above are correct ?

(a) I and 2 only

(b) 2 and 3 only

(c) 1 and 3 only

(d) 1, 2 and 3

Show Answer

Ans: (d)

Explanation

Perfluoroalkyl and polyfluoroalkyl substances (PFAS) are synthetic chemicals used in various consumer products for their water and grease resistance properties.

Statement I is correct: PFAS are indeed widespread contaminants that have been detected in drinking water sources, various food items (due to food packaging materials), and even in the environment. Their widespread use and persistence have led to global contamination of these substances.

Statement II is correct: PFAS are known for their persistence in the environment. They are highly resistant to degradation due to their strong carbon-fluorine bonds. This resistance to breakdown in natural processes contributes to their long-term presence in the environment and their ability to bioaccumulate in living organisms.

Statement III is correct: Due to their persistence, PFAS can accumulate in organisms over time, a process known as bioaccumulation. Once accumulated, PFAS can biomagnify through food chains, potentially reaching higher concentrations in top predators compared to lower trophic levels.


88. Consider the following :

1. Carabid beetles

2. Centipedes

3. Flies

4. Termites

5. Wasps

Parasitoid species are found in how many of the above kind Of organisms ?

(a) Only two

(b) Only three

(c) Only four

(d) All five

Show Answer

Ans: (a)

Explanation

Parasitoid species are organisms that lay their eggs on or inside another organism (the host), ultimately leading to the death of the host as the parasitoid larvae develop.

Carabid beetles: Carabid beetles are predatory insects that do not typically have parasitoid species associated with them. They hunt and consume other invertebrates but are not known for being hosts to parasitoids.

Centipedes: Centipedes are predatory arthropods that also do not typically host parasitoid species. They hunt and feed on various smaller invertebrates but do not exhibit parasitoid behavior.

Flies: Flies (order Diptera) include many species that can be hosts to parasitoid species. For example, various species of parasitic wasps (e.g., Tachinidae, Ichneumonidae) lay their eggs on or inside fly hosts, where the parasitoid larvae develop.

Termites: Termites are social insects that are not typically hosts for parasitoid species. They primarily feed on wood and plant materials within their colonies and are not known for being parasitized by other organisms.

Wasps: Wasps (order Hymenoptera) include many species that are parasitoids themselves. Examples include ichneumon wasps and braconid wasps, which lay their eggs on or inside other insects (including flies) as hosts for their larvae.


89. Consider the following plants :

1. Groundnut

2. Horse-gram

3. Soybean

How many of the above belong to the pea family?

(a) Only one

(b) Only two

(c) All three

(d) None

Show Answer


Ans: (c)

Explanation

All three plants listed — Groundnut (peanut), Horse-gram (horse gram), and Soybean — belong to the pea family, scientifically known as Fabaceae or Leguminosae. This family is characterized by plants that typically produce fruits in the form of pods.

Groundnut (Peanut): Arachis hypogaea belongs to the Fabaceae family. It is known for its edible seeds that develop underground.

Horse-gram: Macrotyloma uniflorum, commonly known as horse gram, is also a member of the Fabaceae family. It is cultivated for its seeds, which are used as a food crop in various regions.

Soybean: Glycine max is another member of the Fabaceae family. It is a major crop grown for its edible beans, which are high in protein and oil content.

Fabaceae is a large family of flowering plants, commonly known as legumes, which includes many important agricultural crops due to their ability to fix nitrogen in the soil and their nutritional value.

Therefore, all three plants listed in the question — Groundnut, Horse-gram, and Soybean — belong to the pea family (Fabaceae or Leguminosae).


90. Consider the following statements :

Statement-I

The Indian Flying Fox is placed under the “vermin” category in the Wild Life (Protection) Act, 1972.

Statement- II:

The Indian Plying Fox feeds on the blood Of Other animals.

Which one Of the following is correct in respect of the above statements ?

(a) Both Statement-I and Statement-II are correct and Statement-II explains

Statement-I

(b) Both Statement-I and Statement-U are correct, but Statement-II does not explain Statement-I

(c) Statement-I is correct, but Statement-II is incorrect

(d) Statement - I is incorrect, but Statement-II is correct

Show Answer

Ans: (c)

Explanation

Statement I is correct: In India, the Indian Flying Fox (Pteropus giganteus) is indeed classified as vermin under Schedule V of the Wildlife (Protection) Act, 1972. This classification allows for certain management measures to control its population in specific circumstances, primarily due to perceived agricultural damage.

Statement-II is correct: The Indian Flying Fox is a frugivorous bat, meaning it primarily feeds on fruits, nectar, and flowers. It does not consume blood; that behavior is typically associated with vampire bats, which are not found in India.


91. Consider the following statements :

Statement-I :

The atmosphere is heated more by incoming solar radiation than by terrestrial radiation,

Statement-II :

Carbon dioxide and Other greenhouse gases in the atmosphere are good absorbers of long

wave radiation.

Which one of the following is correct in respect of the above statements ?

(a) Both Statement-I and Statement-II are correct and Statement-II explains Statement-I

(b) Both Statement-I and Statement-II are correct, but Statement-II does not explain Statement-I

(c) Statement-I is correct, but Statement-II is incorrect

(d) Statement-I is incorrect, but Statement-II is correct

Show Answer

Ans: (d)

Explanation

Statement-I is incorrect: While incoming solar radiation (shortwave radiation) heats the Earth's surface directly, a significant portion of the atmosphere's heating actually comes from the absorption of longwave radiation (terrestrial radiation) emitted by the Earth's surface. The atmosphere absorbs some of this outgoing longwave radiation, which contributes to its heating

Statement-II is correct: Greenhouse gases like carbon dioxide, water vapour, methane, and others have the ability to absorb and re-emit longwave radiation (infrared radiation). This absorption of longwave radiation by greenhouse gases is crucial in the greenhouse effect, where they trap some of the heat energy and keep the Earth's surface warmer than it would be otherwise.


92. ' Consider the following statements :

Statement-I :

Thickness of the troposphere at the equator is much greater as compared to poles.

Statement-II :

At the equator, heat is transported to great heights by strong convectional currents.

Which one of the following is  correct in respect of the above statements ?

(a) Both Statement-I and Statement-II are correct and Statement-II explains

Statement-I

(b) Both Statement-I and Statement-II are correct, but Statement-II does not

explain Statement-I

(c) Statement-I is correct, but Statement-II is incorrect

(d) Statement-I is incorrect, but Statement-II is correct

Show Answer

Answer: (a)


93. Consider the following :

I. Pyroclastic debris

2. Ash and dust

3. Nitrogen

4. Sulphur compounds

How many of the above are products of volcanic eruptions ?

(a) Only one

(b) Only two

(c) Only three

(d) All four

Show Answer

Ans: (a)

Explanation

Statement-I is correct: The troposphere, which is the lowest layer of the Earth's atmosphere where most weather occurs, is thicker at the equator compared to the poles. This is primarily due to the warmer temperatures at the equator, which lead to greater vertical expansion of the atmosphere.

Statement-II is also correct: Near the equator, the Sun's energy is more concentrated, leading to intense heating of the Earth's surface. This heating creates strong upward convection currents (also known as convectional currents) where warm air rises rapidly to high altitudes. These convection currents transport heat energy vertically through the troposphere, contributing to the thicker troposphere observed at the equator.


94. Which of the following is/are correct inference/inferences from isothermal maps in the month of January ?

1. The isotherms deviate to the north over the ocean and to the south over. The continent.

2. The presence Of cold ocean currents, Gulf Stream and North Atlantic Drift make the North Atlantic Ocean colder and the isotherms bend towards the north.

Select the answer using the code given below :

(a) 1 only

(b) 2 only

(c) Both 1 and 2

(d) Neither 1 nor 2

Show Answer

Ans: (d)

Explanation

Pyroclastic Debris: Pyroclastic debris refers to a type of material ejected during volcanic eruptions. It consists of fragments ranging in size from fine ash to large blocks and bombs. Pyroclastic debris is formed when magma is violently expelled from a volcano, and it solidifies into various sizes and shapes as it cools rapidly in the air.

Ash and dust: Volcanic eruptions produce fine ash particles and dust that are carried high into the atmosphere. These particles can travel long distances and affect weather patterns and air quality.

Nitrogen: While nitrogen itself is not typically a direct product of volcanic eruptions, volcanic activity can release gases that include nitrogen compounds. These gases can contribute to the formation of aerosols and other atmospheric effects.

Sulphur compounds: Volcanic eruptions release sulphur dioxide (SO2) and other sulphur compounds into the atmosphere. These compounds can form sulphur aerosols and contribute to atmospheric cooling and acid rain when they react with water vapour.


95. Which of the following countries are well known as the two largest cocoa producers in the

world ?

(a) Algeria and Morocco

(b) Botswana and Namibia

(c) Cöte d'Ivoire and Ghana

(d) Madagascar and Mozambique

Show Answer

Ans: (c)

Explanation

Côte d’Ivoire (Ivory Coast) and Ghana are the top two cocoa-producing countries globally.

These countries have favourable climates and soil conditions for cocoa cultivation, and their economies heavily rely on cocoa production and export.

Together, they account for a significant majority of the world’s cocoa supply.

96. With reference to the Himalayan rivers joining the Ganga downstream of Prayagraj

from West to East, which one of the following sequences is correct ?

(a) Ghaghara — Gomati — Gandak — Kosi

(b). Gomati — Ghaghara — Gandak — Kosi

(c) Ghaghara — Gomati — Kosi — Gandak

(d) Gomati — Ghaghara — Kosi — Gandak

Show Answer

Ans: (b)

Explanation

Statement-I is correct: Rainfall contributes to weathering processes in several ways. It physically erodes rocks and minerals through the action of raindrops and running water. Additionally, rainwater can chemically weather minerals by dissolving them or reacting with them when it contains acidic components.

Statement-II is correct : Rainwater naturally absorbs carbon dioxide (CO2) from the atmosphere as it falls through the air. This dissolved CO2 forms carbonic acid (H2CO3) in rainwater, which can enhance chemical weathering of minerals.

Statement-III is correct: Rain is due to condensation of water droplets in the atmosphere. It does not contain any dissolved salts like ordinary water. Therefore rain water can dissolve more amount of oxygen than ordinary water. Moreover rain water is in contact with atmospheric air. Hence, it contains more percentae of oxygen.

97. Consider the following statements :

Statement-I :

Rainfall is one Of the reasons for weathering of rocks

Statement-II :

Rain water contains carbon dioxide in solution.

Statement-III :

Rain water contains atmospheric oxygen.

Which one of the following is correct in respect of the above statements ?

(a) Both Statement-II and Statement-III are correct and both of them explain

Statement-I

(b) Both Statement-II and Statement-III are correct, but only one of them explains Statement-I

(c) Only one of the Statements II and III is correct and that explains Statement-I

(d) Neither Statement-II nor Statement-III Is correct.

Show Answer

Ans: (b)

Explanation

Statement-I is correct: Rainfall contributes to weathering processes in several ways. It physically erodes rocks and minerals through the action of raindrops and running water. Additionally, rainwater can chemically weather minerals by dissolving them or reacting with them when it contains acidic components.

Statement-II is correct : Rainwater naturally absorbs carbon dioxide (CO2) from the atmosphere as it falls through the air. This dissolved CO2 forms carbonic acid (H2CO3) in rainwater, which can enhance chemical weathering of minerals.

Statement-III is correct: Rain is due to condensation of water droplets in the atmosphere. It does not contain any dissolved salts like ordinary water. Therefore rain water can dissolve more amount of oxygen than ordinary water. Moreover rain water is in contact with atmospheric air. Hence, it contains more percentae of oxygen.

98. Consider the following countries :

1. Finland

2. Germany

3. Norway

4. Russia

How many of the above countries have a border with the North Sea ?

(a) Only one

(b) Only two

(c) Only three

(d) All four

Show Answer

Ans: (b)

Explanation

Germany and Norway

99. Consider the following information :

S.No Waterfall Region River
1 Dhuandhar Malwa Narmada
2 Hundru Chota Nagpur Subarnarekha
3 Gersoppa Western Ghats Netravati

In how many of the rows is the given information correctly matched ?

(a) Only one

(b) Only two

(c) All three

(d) None

Show Answer

Ans: (a)

Explanation

Hundru Falls: Location: Ranchi district, Chotanagpur region, Jharkhand, On Subarnarekha river

The famous Jog Falls is in Karnataka state. Jog Falls is also known as Gersoppa fall. Jog Falls was formed by the Sharavati river.

Dhuandhar fall in Jabalpur which lies in Mahakoshal region


100. Consider the following information :

S.No Region Name of the mountain range Type of mountain
1 Central Asia Vosges Fold mountain
2 Europe Alps Block mountain
3 North America Appalachians Fold mountain
4 South America Andes Fold mountain

In how many of the above rows is the given information correctly matched ?

(a) Only one

(b) Only two

(c) Only three

(d) All four

Show Answer

Ans: (b)

Explanation

Row I is incorrect. The Vosges Mountains are located in eastern France, not in Central Asia. Additionally, the Vosges are an example of block mountains, not fold mountains. Row II is incorrect. The Alps are indeed located in Europe, but they are an example of fold mountains, not block mountains.

Row III is correct. The Appalachian Mountains are located in eastern North America and are an example of old fold mountains.

Row IV is correct. The Andes Mountains run along the western coast of South America and are an example of young fold mountains

Get in Touch

Bringing Your Vision To Life

View Details
- +
Sold Out